Inifinity limit with natural log

Click For Summary
SUMMARY

The limit of the expression Limx→∞ Ln(x2-1) - Ln(2x2+3) simplifies to Ln(1/2) or equivalently -Ln(2). The correct approach involves applying the properties of logarithms to combine the terms into a single logarithmic expression. By dividing both the numerator and denominator by x2, the limit of the fraction inside the logarithm approaches 1/2 as x approaches infinity, leading to the final result.

PREREQUISITES
  • Understanding of limits in calculus
  • Familiarity with logarithmic properties
  • Basic algebraic manipulation skills
  • Knowledge of asymptotic behavior of functions
NEXT STEPS
  • Study the properties of logarithms in calculus
  • Learn about limits involving logarithmic functions
  • Explore asymptotic analysis techniques
  • Practice solving limits with polynomial expressions
USEFUL FOR

Students studying calculus, particularly those focusing on limits and logarithmic functions, as well as educators looking for examples of limit evaluations involving natural logarithms.

AlexandraMarie112
Messages
16
Reaction score
1

Homework Statement


Limx--> ∞ Ln(x^2-1) -Ln(2x^2+3)

Homework Equations

The Attempt at a Solution


Ln(x^2-1)/(2x^2+3)

Then I divided the top and bottom by x^2 so in the end I got (1/2).

Is this right?
 
Physics news on Phys.org
AlexandraMarie112 said:

Homework Statement


Limx--> ∞ Ln(x^2-1) -Ln(2x^2+3)

Homework Equations

The Attempt at a Solution


Ln(x^2-1)/(2x^2+3)

Then I divided the top and bottom by x^2 so in the end I got (1/2).

Is this right?
What happened to the ##\ln##?
 
  • Like
Likes Mastermind01
Is this what you did? :

##\lim_{n\rightarrow +\infty} {\ln {(x^2 - 1)} - \ln{(2x^2+3)}}##

## = \lim_{n\rightarrow +\infty} {\ln ({\frac{x^2 - 1}{2x^2+3}})}#### = {\ln {\lim_{n\rightarrow +\infty}(\frac{1 - \frac{1}{x^2}}{2+\frac{3}{x^2}}})}##

You got the limit of the inside part as ##\frac{1}{2}## you need to take its ##\ln## to get the right answer.
 
Mastermind01 said:
Is this what you did? :

##\lim_{n\rightarrow +\infty} {\ln {(x^2 - 1)} - \ln{(2x^2+3)}}##

## = \lim_{n\rightarrow +\infty} {\ln ({\frac{x^2 - 1}{2x^2+3}})}#### = {\ln {\lim_{n\rightarrow +\infty}(\frac{1 - \frac{1}{x^2}}{2+\frac{3}{x^2}}})}##

You got the limit of the inside part as ##\frac{1}{2}## you need to take its ##\ln## to get the right answer.
Yes that's what I did. So my final answer then should be Ln(1/2) ?
 
AlexandraMarie112 said:
Yes that's what I did. So my final answer then should be Ln(1/2) ?
Right, or -ln(2)
 
  • Like
Likes Mastermind01 and AlexandraMarie112
Question: A clock's minute hand has length 4 and its hour hand has length 3. What is the distance between the tips at the moment when it is increasing most rapidly?(Putnam Exam Question) Answer: Making assumption that both the hands moves at constant angular velocities, the answer is ## \sqrt{7} .## But don't you think this assumption is somewhat doubtful and wrong?

Similar threads

  • · Replies 6 ·
Replies
6
Views
2K
  • · Replies 5 ·
Replies
5
Views
1K
Replies
5
Views
2K
  • · Replies 9 ·
Replies
9
Views
1K
  • · Replies 1 ·
Replies
1
Views
1K
  • · Replies 4 ·
Replies
4
Views
1K
  • · Replies 7 ·
Replies
7
Views
4K
Replies
5
Views
2K
  • · Replies 4 ·
Replies
4
Views
1K
  • · Replies 4 ·
Replies
4
Views
2K